Difference between revisions of "1966 AHSME Problems/Problem 39"

(Created page with "== Problem == In base <math>R_1</math> the expanded fraction <math>F_1</math> becomes <math>.373737\cdots</math>, and the expanded fraction <math>F_2</math> becomes <math>.737373...")
 
(Solution)
Line 5: Line 5:
  
 
== Solution ==
 
== Solution ==
 
+
<math>\fbox{E}</math>
  
 
== See also ==
 
== See also ==

Revision as of 02:36, 15 September 2014

Problem

In base $R_1$ the expanded fraction $F_1$ becomes $.373737\cdots$, and the expanded fraction $F_2$ becomes $.737373\cdots$. In base $R_2$ fraction $F_1$, when expanded, becomes $.252525\cdots$, while the fraction $F_2$ becomes $.525252\cdots$. The sum of $R_1$ and $R_2$, each written in the base ten, is:

$\text{(A) } 24 \quad \text{(B) } 22 \quad \text{(C) } 21 \quad \text{(D) } 20 \quad \text{(E) } 19$

Solution

$\fbox{E}$

See also

1966 AHSME (ProblemsAnswer KeyResources)
Preceded by
Problem 38
Followed by
Problem 40
1 2 3 4 5 6 7 8 9 10 11 12 13 14 15 16 17 18 19 20 21 22 23 24 25 26 27 28 29 30
All AHSME Problems and Solutions

The problems on this page are copyrighted by the Mathematical Association of America's American Mathematics Competitions. AMC logo.png